LSAT and Law School Admissions Forum

Get expert LSAT preparation and law school admissions advice from PowerScore Test Preparation.

 Administrator
PowerScore Staff
  • PowerScore Staff
  • Posts: 8917
  • Joined: Feb 02, 2011
|
#72655
Complete Question Explanation

Justify the Conclusion, SN. The correct answer choice is (E).

The argument in the stimulus contains a missing link, and the correct answer choice must supply that link in order to justify the conclusion. The conclusion is a conditional statement that if ad agencies target older adults with their advertisements, this will maximize their clients' profits. The basis for this conditional is a premise about older adults' level of personal disposable income: they have more of it than the rest of the population combined. The conclusion contains "new information," in the form of the reference to "maximizing clients' profits." We therefore need to bridge the gap between targeting a group that controls the level of disposable income controlled by older adults, and maximizing clients' profits. Put differently, we're missing information about whether targeting older adults with advertisements would actually cause them to spend enough money to maximize the ad agencies' clients' profits.

Answer choice (A): Without referring to the "new information" in the conclusion ("maximizing clients' profits"), an answer choice cannot justify the conclusion. Since answer choice A does not refer to maximizing client profits (even in a synonymous way), answer choice A cannot be the correct answer.

Answer choice (B): Answer choice B is the Mistaken Reversal of what we need to justify the conclusion. In other words, we do not want an extra premise that tells us targeting older adults is necessary for maximizing clients' profits, as answer choice B does (note the "unless"). Rather, we want an answer choice that tells us targeting such adults is sufficient for maximizing clients' profits (in which case, we would know our premises would be sufficient to prove the conclusion).

Answer choice (C): Answer choice C refers to the wrong targets of the advertisements ("wealthiest people"). In the argument, older people are not necessarily the wealthiest people (as individuals). Rather, they are the people who (as a group) control the most disposable personal income. Further, answer choice C refers to the wrong goal: "improve the reputation" of products, rather than the goal in the stimulus of maximizing profits. Put differently, answer choice C also fails for the same reason answer choice A failed (no reference to maximizing client profits).

Answer choice (D): Answer choice D is the Mistaken Negation of what we need to prove the conclusion. It says (rephrased) that if an advertising agency targets people who do NOT control as much disposable income, it will NOT maximize its clients' profits. Since a Mistaken Negation is never something we can infer with certainty, answer choice D cannot fully justify the conclusion.

Answer choice (E): This is the correct answer choice. Answer choice E fixes the conditional mistakes made by answer choices B and D. It says that if an advertising agency does target people who do (as a group) control the most disposable income (i.e. the older adults referenced in the premises of the stimulus), then it will (necessarily) maximize its clients' profits. This is exactly the relationship we want in a conditional answer choice in Justify questions. We want the sufficient condition to fully match the premises of the stimulus (because then the premises will be sufficient, i.e. enough). We want the necessary condition to fully match the conclusion of the stimulus (because then the conclusion will be something that is certain, i.e. necessary).
 ShannonOh22
  • Posts: 70
  • Joined: Aug 15, 2019
|
#73192
Can you please explain the difference between answer choice B and answer choice E? It seems to me they are saying the same thing, with the one slight exception that E specifies "50% or more" instead of saying "most", as B does. In LSAT speak, aren't these quantifiers synonymous?

There are also a lot of confusing negatives and double negatives, so I'm sure I mixed up my diagramming somehow. A conditional diagram breakdown of each answer choice would be greatly appreciated!
 dlehr99
  • Posts: 24
  • Joined: Dec 06, 2019
|
#73222
Shannon -I'll give this a shot as you may be studying for Monday's exam like me. I also had it down to these two answers. I believe the differentiation is that answer (B) is focused on the companies' actions to maximize profits whereas answer (D) is focused, correctly, on the Advertising agencies' actions. The conclusion in the stimulus states "Therefore, advertising agencies can maximize" meaning we need to provide an assumption that can be associated with them not the companies. From the stimulus it would be impossible to know if companies have alternatives to maximizing profits, whereas we do know that the Ad agencies do not have another alternative.

... At least to this (clearly misinformed) Social Observer as it's pretty widely accepted its much more difficult to change the preferences of that age group. Nonetheless...
 Robert Carroll
PowerScore Staff
  • PowerScore Staff
  • Posts: 1787
  • Joined: Dec 06, 2013
|
#73368
Shannon,

Answer choice (B) is the Mistaken Reversal of what we're looking for.

Use the Unless Equation. "Unless" modifies the necessary condition. Let's abbreviate that condition as "markets to a population with a majority of income". The other condition must be negated to become the sufficient condition. So everything before the "unless" gets negated to become sufficient. Let's abbreviate it "company maximizes profits".

We have then:

company maximizes profits :arrow: markets to a population with a majority of income

We have a Justify question here. Saying that this strategy is NECESSARY for maximizing profits does not make it SUFFICIENT to do so.

Answer choice (E) is essentially the reversal of this, and that makes the strategy sufficient to maximize profits, like we want.

Note that there is no need to focus on advertising agencies for this Justify question. Answer choice (E) would have been ok if it had applied to all companies, because, in a Justify, an answer that's broader than necessary is still ok as long as it proves the conclusion. For an Assumption question, overbreadth is terrible for an answer.

Robert Carroll
 vbkehs
  • Posts: 31
  • Joined: Mar 31, 2020
|
#74663
Can you please explain why D is wrong here?
 Jeremy Press
PowerScore Staff
  • PowerScore Staff
  • Posts: 1000
  • Joined: Jun 12, 2017
|
#74679
Hi vbkehs,

We've just posted a complete explanation to this question above. Please let us know if that resolves your question about answer choice D. Thanks!

Jeremy
User avatar
 simonsap
  • Posts: 34
  • Joined: Jun 14, 2021
|
#88479
premise: old people = 50%+ income controllers
conclusion: Advertise to Old -- > max profit
missing link?
Advertise to 50%+ income controllers --> max profit

put differently:
X=Y
X therefore Z

missing link?
Y therefore Z



A cool tip here is seeing all the permutations a SA question can be constructed.
A simple logical reasoning chain looks like this:
A-->B-->C

A SA question can take out any of the links.

A stimulus can independently link premises to a conclusion without linking the premises together:
A-->C
B-->C
here you have to link A to B

B-->C
A-->C
again, you have to link A to B

Or a stimulus can link the premises together without connecting them to the conclusion
A-->B
C
Here you have to link B to C

Try it with a real example:
p1. All greyhounds are dogs (Greyhound --> Dog)
p2. All dogs have 4 legs (Dog ---> 4 legs)
cc. All greyhounds have 4 legs (Greyhound --> 4 legs)

G --> D --> 4L

G-->D
D-->4L
G-->4L

Again, any of these links can be removed to construct a SA question.



Question 20 has two challenges.
1) translating/realizing that a group that controls more than the rest of the population is 50%+
2) the chain:m
Old people --> Control 50% of income (so advertise to them)--> maximize profits

the stimulus gives us:
old people --> control 50% of income (A -->B)
old people --> maximize profits (B -->C)

answer E correctly connects the dots:
control 50% of income --> maximize profits (A -->B)

answer B essentially produces an incorrect answer by doing the opposite
maximize profits --> control 50% of income (C -->B)

Get the most out of your LSAT Prep Plus subscription.

Analyze and track your performance with our Testing and Analytics Package.